subject
Mathematics, 04.04.2020 10:54 melanyrivera776

If m ≤ f(x) ≤ M for a ≤ x ≤ b, where m is the absolute minimum and M is the absolute maximum of f on the interval [a, b], then m(b − a) ≤ b f(x) dx a ≤ M(b − a). Use this property to estimate the value of the integral. π/6 5 tan(2x) dx π/8

ansver
Answers: 1

Another question on Mathematics

question
Mathematics, 21.06.2019 17:00
What is the value of the expression 9+n/3-6 when n = 12? 1. 5. 7. 12.
Answers: 1
question
Mathematics, 21.06.2019 20:00
Can somebody 1. what is the formula for finding the vertical distance between two points on a coordinate plane?
Answers: 3
question
Mathematics, 21.06.2019 22:10
Rationalize the denominator- 12x/√x-10
Answers: 1
question
Mathematics, 21.06.2019 23:20
The graph of y=x3 is transformed as shown in the graph below. which equation represents the transformed function?
Answers: 1
You know the right answer?
If m ≤ f(x) ≤ M for a ≤ x ≤ b, where m is the absolute minimum and M is the absolute maximum of f on...
Questions
question
Mathematics, 10.11.2019 05:31
Questions on the website: 13722367